Page 8 of 13

Re: LSAT Q&A: Ask the Experts

Posted: Tue May 29, 2012 3:28 pm
by cc.celina
Hi experts, thanks so much for agreeing to do this!

No matter how much I stare at PT 53, Sec 3, Q 22 I can't reason out why B is the answer.

Since this is a explains-the-paradox-except question, I reasoned that 4 of the answer choices should explain both why a) books read went down and b) profits up, or at least explain one while allowing the other to plausibly exist. I eliminated C, D, E pretty quickly.

It was down to A and B, both of which I thought were shaky explanations, but I ended up (incorrectly) picking A because I thought it required more mental acrobatics to explain the paradox, and while it clearly explains lower reading levels, I didn't see a clear connection to how it explains bookstore profits increasing. (I've seen a few explanations that suggest that the people who would ordinarily read library books are now being forced to buy from bookstores -- OK, while it seems like a stretch, I'll accept it, but then how does it explain reading levels decreasing?? If people are just switching where they get their books from, wouldn't that mean the number of books read would remain the same?) For me, it required the very shaky assumption that SOME people transferred to bookstores for their reading needs while SOME simply stopped reading that genre altogether because they no longer had access at the library, and I've been oft-warned against making stretchy assumptions like that.

I figured B was a (more) plausible explanation. This may seem like silly reasoning, but while I was taking the test, it didn't seem any sillier than what A was calling for: If shoplifting was such a problem that they had to install high-end antitheft stuff, a lot of people in the past must have been stealing books from the store, i.e. causing reading levels to go up while profits went down (presumably significantly, since there was a noticeable difference between bookstores and other industries when the increase happened). The antitheft equipment, which insulated the bookstores from the rise in shoplifting in general, would therefore discourage shoplifters (causing them to read less) and increase company profits.

It was pointed out to me that I'd have to make the assumption that shoplifting actually occurred in bookstores prior to the antitheft install for my reasoning to be sound. This didn't seem like a stretch - why would there be a "recent" install that randomly occurred in all bookstores around the same time, if it theft wasn't a problem? You don't fix things that ain't broke.

Neither of them seem like a good answer to me, but B seems to make more solid sense in that it can account for BOTH sides of the paradox instead of just one or the other. Help! 99% of LR questions make perfect sense to me if I take the time, but when I miss my target on PTs, its often because of one of these LR questions that I just can't justify even if I stare at it for an hour.

And more generally, what do you do when you're down to 2 answer choices and you feel like neither is correct? How far do assumptions go? Am I fundamentally misunderstanding paradox questions?

Re: LSAT Q&A: Ask the Experts

Posted: Tue May 29, 2012 4:19 pm
by Manhattan LSAT Noah
cc.celina wrote:Hi experts, thanks so much for agreeing to do this!

No matter how much I stare at PT 53, Sec 3, Q 22 I can't reason out why B is the answer.

Since this is a explains-the-paradox-except question, I reasoned that 4 of the answer choices should explain both why a) books read went down and b) profits up, or at least explain one while allowing the other to plausibly exist. I eliminated C, D, E pretty quickly.

It was down to A and B, both of which I thought were shaky explanations, but I ended up (incorrectly) picking A because I thought it required more mental acrobatics to explain the paradox, and while it clearly explains lower reading levels, I didn't see a clear connection to how it explains bookstore profits increasing. (I've seen a few explanations that suggest that the people who would ordinarily read library books are now being forced to buy from bookstores -- OK, while it seems like a stretch, I'll accept it, but then how does it explain reading levels decreasing?? If people are just switching where they get their books from, wouldn't that mean the number of books read would remain the same?) For me, it required the very shaky assumption that SOME people transferred to bookstores for their reading needs while SOME simply stopped reading that genre altogether because they no longer had access at the library, and I've been oft-warned against making stretchy assumptions like that.

I figured B was a (more) plausible explanation. This may seem like silly reasoning, but while I was taking the test, it didn't seem any sillier than what A was calling for: If shoplifting was such a problem that they had to install high-end antitheft stuff, a lot of people in the past must have been stealing books from the store, i.e. causing reading levels to go up while profits went down (presumably significantly, since there was a noticeable difference between bookstores and other industries when the increase happened). The antitheft equipment, which insulated the bookstores from the rise in shoplifting in general, would therefore discourage shoplifters (causing them to read less) and increase company profits.

It was pointed out to me that I'd have to make the assumption that shoplifting actually occurred in bookstores prior to the antitheft install for my reasoning to be sound. This didn't seem like a stretch - why would there be a "recent" install that randomly occurred in all bookstores around the same time, if it theft wasn't a problem? You don't fix things that ain't broke.

Neither of them seem like a good answer to me, but B seems to make more solid sense in that it can account for BOTH sides of the paradox instead of just one or the other. Help! 99% of LR questions make perfect sense to me if I take the time, but when I miss my target on PTs, its often because of one of these LR questions that I just can't justify even if I stare at it for an hour.

And more generally, what do you do when you're down to 2 answer choices and you feel like neither is correct? How far do assumptions go? Am I fundamentally misunderstanding paradox questions?
I think you want to focus more on finding an answer (in this case incorrect since it's an EXCEPT question) that supports/explains the existence of both facts at the same time. It's not that both facts need to be explained, it's more that we need an answer that leaves us thinking, "oh, now I see how it can be both that ___ and ____."

(A) does this. How is it that bookstores are seeing increased profits even though people are reading less? Because libraries aren't providing the reading material, so folks are heading to those bookstores in enough numbers to overcome the slowdown in reading.

With (B), we have an explanation for why profits might not have dropped--the recent wave of shoplifting hasn't hit bookstores--but we don't see why they'd make more profit in this reading downturn.

(And, to respond to one of your points, notice that the shoplifting is a recent increase. The gymnastics you have to do to get the shoplifting to have been affecting bookstores in the past three years are significant--isn't it just as easy to say that shoplifting has been going on forever and it's only now that the technology is available?)

I hope that helps -- happy to keep the conversation going.

Re: LSAT Q&A: Ask the Experts

Posted: Tue May 29, 2012 4:39 pm
by BalanceCare
Hi Experts. I have a couple of principle questions I'd like your help with, if you don't mind.

PT 54, LR1, #18
P: [Strong Position] –Likely--> [Misinterpret Evidence]
P: [Understand] --> [Consider Impartially]
C: [Strong Position] --> [Already considered conflicting evidence]
I am having trouble understanding exactly what the question wants from me. I know that C is the right answer; this leads me to wonder, will the correct answer choice to an “identify the principle” question always be the conclusion or its contrapositive?


PT63, LR1 #10:

I picked the right answer, but something about it seems weird. (I think this is similar to a question that was recently asked but I’m still having trouble with the concept so please excuse the repetition.)
It seems that the argument consists of:
P: [Ms. S’s actions led to damage]
A: [If you could reasonably expect that your action would cause damage] --> [Pay for the damage]
C: [If Ms. S could reasonably expect that action would lead to damage] --> [She should pay for the damage]
As I see it, the conclusion is itself a conditional principal, and this seems odd. It seems like we should be trying to find out whether or not Ms. S had a reasonable expectation of the damage or not, as this will determine whether or not she should pay. I see that A, the correct answer, connects the premise to the conclusion, but the whole thing just seems weird to me. Can anyone illuminate this?

Re: LSAT Q&A: Ask the Experts

Posted: Tue May 29, 2012 4:58 pm
by Manhattan LSAT Noah
BalanceCare wrote: PT62, LR1 #10:

I picked the right answer, but something about it seems weird. (I think this is similar to a question that was recently asked but I’m still having trouble with the concept so please excuse the repetition.)
It seems that the argument consists of:
P: [Ms. S’s actions led to damage]
A: [If you could reasonably expect that your action would cause damage] --> [Pay for the damage]
C: [If Ms. S could reasonably expect that action would lead to damage] --> [She should pay for the damage]
As I see it, the conclusion is itself a conditional principal, and this seems odd. It seems like we should be trying to find out whether or not Ms. S had a reasonable expectation of the damage or not, as this will determine whether or not she should pay. I see that A, the correct answer, connects the premise to the conclusion, but the whole thing just seems weird to me. Can anyone illuminate this?
I believe this is not the right question numbering. Can you confirm?

Re: LSAT Q&A: Ask the Experts

Posted: Tue May 29, 2012 5:08 pm
by BalanceCare
Manhattan LSAT Noah wrote: I believe this is not the right question numbering. Can you confirm?
Doh! PT 63, not 62. Apologies for the goose-chase. Edited the original to fix. Thanks!

Re: LSAT Q&A: Ask the Experts

Posted: Tue May 29, 2012 5:12 pm
by Manhattan LSAT Noah
BalanceCare wrote:Hi Experts. I have a couple of principle questions I'd like your help with, if you don't mind.

PT 54, LR1, #18
P: [Strong Position] –Likely--> [Misinterpret Evidence]
P: [Understand] --> [Consider Impartially]
C: [Strong Position] --> [Already considered conflicting evidence]
I am having trouble understanding exactly what the question wants from me. I know that C is the right answer; this leads me to wonder, will the correct answer choice to an “identify the principle” question always be the conclusion or its contrapositive?
This is a particularly tricky question because that first sentence is a distraction.

To your final question, no. Usually, the answer to these principle support questions is the argument's assumption (this is different than principle example questions, in which we're given the principle). In this case, we have one premise to consider and a conclusion:

P: Understand --> consider evidence
C: Strong --> consider evidence.

The assumption here is obviously that strong --> understand. And, (C) gives us the contrapositive: - understand --> - strong.

I hope that helps.

Re: LSAT Q&A: Ask the Experts

Posted: Tue May 29, 2012 5:18 pm
by Manhattan LSAT Noah
BalanceCare wrote: PT63, LR1 #10:

I picked the right answer, but something about it seems weird. (I think this is similar to a question that was recently asked but I’m still having trouble with the concept so please excuse the repetition.)
It seems that the argument consists of:
P: [Ms. S’s actions led to damage]
A: [If you could reasonably expect that your action would cause damage] --> [Pay for the damage]
C: [If Ms. S could reasonably expect that action would lead to damage] --> [She should pay for the damage]
As I see it, the conclusion is itself a conditional principal, and this seems odd. It seems like we should be trying to find out whether or not Ms. S had a reasonable expectation of the damage or not, as this will determine whether or not she should pay. I see that A, the correct answer, connects the premise to the conclusion, but the whole thing just seems weird to me. Can anyone illuminate this?
You're right that the conclusion is a relationship. Usually we see this in sufficient assumption questions, but hey, here we are.

You have the argument structure down nicely. And, this is another principle support question, so expect the answer to be the assumption. As to your point about what it seems like we should be trying to find, we don't need to prove that the conditional conclusion gets triggered, we just need to show that it is logically valid.

Similarly, if I asked you to support this: If people stop smoking they live a month longer. You don't need to support whether people actually stop smoking. You just need to show that if they do, they'll live a month longer.

I hope that clears it up.

Re: LSAT Q&A: Ask the Experts

Posted: Wed May 30, 2012 1:46 pm
by shifty_eyed
PT 60 Section 1 Q20

I chose E because I thought since we know radio drama --> thinking about what you hear ---> exercising imagination that an assumption required would be that television ---> not thinking about what you see ---> not exercising imagination.

I eliminated D because nothing seemed too strong because the conclusion only states that tv viewers use imagination less frequently

Please advise!

Re: LSAT Q&A: Ask the Experts

Posted: Wed May 30, 2012 2:01 pm
by shifty_eyed
PT 60 Section 3 Q 22

I thought answer choice A was a trick answer preying on a shift in language from "responsible" in the stimulus meaning "the cause of" and "bear responsibility" meaning "should be held accountable for". Obviously, I was wrong in interpreting it this way.

B and D I could easily eliminate.

C I first eliminated, and then ended up choosing.
I read this explanation on the MLSAT forum:
(C) is incorrect because the connection between the increased demand and increased price is explicitly stated in the argument. We don't need to assume that increased demand always causes prices to increase; it's enough to know that this is the case in this particular example.

http://www.manhattanlsat.com/forums/q22 ... 92a92d87a6

My line of thought, however, was that if we know that demand always causes prices to increase then the government would have had to know that when putting in place policies that increase consumer demand. Is it safe to say that this wouldn't be sufficient because we still don't know if they knew their policies would increase demand? Or am I on the completely wrong track here?

I would love to hear opinions on this!

Re: LSAT Q&A: Ask the Experts

Posted: Wed May 30, 2012 3:51 pm
by Jeffort
shifty_eyed wrote:PT 60 Section 1 Q20

I chose E because I thought since we know radio drama --> thinking about what you hear ---> exercising imagination that an assumption required would be that television ---> not thinking about what you see ---> not exercising imagination.

I eliminated D because nothing seemed too strong because the conclusion only states that tv viewers use imagination less frequently

Please advise!
RE: (E)

Just because watching TV dramas does not require thinking about the particular images on the screen (since viewers see them) to know what the scenes in the drama look like and don't have to use imagination to render mental pictures of what the scenes in the drama would look like without visual images being provided does not preclude viewers from exercising their imaginations in other ways while watching a show.

When you properly negate (E) logically, it does not exclude other possible ways viewers can, will or need to exercise their imaginations while watching a TV drama show for entertainment.

There are many other things besides what the scenes of the drama look like left to imagination while watching a TV drama. Such as: what are the characters thinking?, what are their motives?, who slept with who?, what's going to happen next?, who committed the crime?, are they going to catch the perpetrator of the crime?, is so and so going to live?, are Castle and Beckett finally going to get together (well, we now know the answer to that one but I've been using my imagination about it for a long time!), etc.

(D) rips apart the argument when negated properly since the argument assumes that consumers of popular entertainment dramas having to visualize the scenes of a drama just from audio is the main factor that has stimulated/caused people to exercise imagination when viewing/listening to them.

The logical force of the word 'nothing' is certainly no reason to eliminate the answer choice. In fact it is part of the reason (D) is the correct answer choice. When negated, you change 'nothing' into 'something' and the argument is destroyed because the evidence provided in support of the conclusion is pretty much rendered useless/irrelevant and the conclusion is left standing alone naked with no logical support.

Re: LSAT Q&A: Ask the Experts

Posted: Wed May 30, 2012 4:14 pm
by shifty_eyed
Jeffort wrote:
shifty_eyed wrote:PT 60 Section 1 Q20

I chose E because I thought since we know radio drama --> thinking about what you hear ---> exercising imagination that an assumption required would be that television ---> not thinking about what you see ---> not exercising imagination.

I eliminated D because nothing seemed too strong because the conclusion only states that tv viewers use imagination less frequently

Please advise!
RE: (E)

Just because watching TV dramas does not require thinking about the particular images on the screen (since viewers see them) to know what the scenes in the drama look like and don't have to use imagination to render mental pictures of what the scenes in the drama would look like without visual images being provided does not preclude viewers from exercising their imaginations in other ways while watching a show.

When you properly negate (E) logically, it does not exclude other possible ways viewers can, will or need to exercise their imaginations while watching a TV drama show for entertainment.

There are many other things besides what the scenes of the drama look like left to imagination while watching a TV drama. Such as: what are the characters thinking?, what are their motives?, who slept with who?, what's going to happen next?, who committed the crime?, are they going to catch the perpetrator of the crime?, is so and so going to live?, are Castle and Beckett finally going to get together (well, we now know the answer to that one but I've been using my imagination about it for a long time!), etc.

(D) rips apart the argument when negated properly since the argument assumes that consumers of popular entertainment dramas having to visualize the scenes of a drama just from audio is the only factor that stimulates people to exercise imagination when viewing/listening to them.

The logical force of the word 'nothing' is certainly no reason to eliminate the answer choice. In fact it is part of the reason (D) is the correct answer choice. When negated, you change 'nothing' into 'something' and the argument is destroyed because the evidence provided in support of the conclusion is pretty much rendered useless/irrelevant and the conclusion is left standing alone naked with no logical support.
Hmm... I guess I need to work on negating answer choices in sufficient assumption questions.

Re: LSAT Q&A: Ask the Experts

Posted: Wed May 30, 2012 4:17 pm
by bp shinners
shifty_eyed wrote: My line of thought, however, was that if we know that demand always causes prices to increase then the government would have had to know that when putting in place policies that increase consumer demand. Is it safe to say that this wouldn't be sufficient because we still don't know if they knew their policies would increase demand? Or am I on the completely wrong track here?

I would love to hear opinions on this!
First off, I have a problem that C doesn't involve responsibility, and that's really where the gap in this argument is. As a practical point, the Necessary Assumption questions are going to deal with the gap/flaw in the argument more often than not, so I would probably skip over C just because I'm really looking for something that talks about assigning responsibility.

C isn't explicitly stated in the argument because C is extremely broad and strong. It states that demand can't increase without price going up. We only know from the argument that that happened as a result of these policies and in this situation. C would have me believe that there's absolutely no way to increase demand without causing an increase in price, which is way too broad and generally applicable, as opposed to being applicable to my situation at hand. Whenever I have an answer that applies to a broader group of situations than I'm talking about, it's not going to be correct for a Necessary Assumption question because I only NEED to assume something that applies in the given case.

Re: LSAT Q&A: Ask the Experts

Posted: Wed May 30, 2012 4:17 pm
by bp shinners
shifty_eyed wrote:
Hmm... I guess I need to work on negating answer choices in sufficient assumption questions.
Don't do this - it only works (reliably at least) for Necessary Assumption questions.

Re: LSAT Q&A: Ask the Experts

Posted: Wed May 30, 2012 4:19 pm
by Jeffort
shifty_eyed wrote:
Hmm... I guess I need to work on negating answer choices in sufficient assumption questions.
NOOOOOOooooooo!!!!!!! Image

Negating answer choices with sufficient assumption questions is NOT a good or logically valid strategy.

Negating answer choices to determine the correct one is a strategy to be applied to necessary assumption questions.

Re: LSAT Q&A: Ask the Experts

Posted: Wed May 30, 2012 4:30 pm
by LSAT Blog
shifty_eyed wrote:Hmm... I guess I need to work on negating answer choices in sufficient assumption questions.
Here's an important reason to know the difference between necessary assumption and sufficient assumption questions when it comes to negation:

http://www.top-law-schools.com/forums/v ... 5#p4933572

Re: LSAT Q&A: Ask the Experts

Posted: Wed May 30, 2012 5:18 pm
by cc.celina
Thanks a bunch for answering my last question. Sadly I have another one: PT 47, Section 1, Q22 (pesticides).

Correct answer is C, but I picked E. How I went about it:

- It's a weaken question, so I noted the conclusion ("US exportation of banned pesticides greatly increases health risk to US consumers") and the assumption required to get to that conclusion: that some pesticides manufactured in the US are significantly harmful to the health US consumers when they subsequently eat products grown with those pesticides.

- Considered C, but I didn't see any reason to believe that, just because other countries manufacture these pesticides, means the ag. products on which they are used are exported to the US in particular. Even if they were, that doesn't render the US-manufactured pesticides any less harmful: the fact that a Smith & Wesson handgun (manufactured in the US) is dangerous to the safety of US citizens isn't changed by the fact that Glocks (manufactured in Austria) are also available. The combination of the two is more dangerous than the sole availability of either one, but that doesn't mean that either taken individually is harmless.
To me, C would only weaken the argument if the columnist had claimed, "Ceasing the practice of manufacturing banned pesticides in the US would completely eliminate the risk posed by banned pesticides." (Just as it would be unreasonable to conclude that, even if Smith & Wesson went out of business tomorrow and all their guns spontaneously self-destructed, nobody in the US would die from guns anymore - because Glocks are available as an alternative.) It didn't seem to have any bearing on the less forceful claim that dangerous US-manufactured pesticides are harmful when consumed.

- I didn't think E was a MUCH better answer, but I thought that it meant that the US wasn't really on top of its banning policy. The stimulus indicates that US banning policy is stricter than other countries, but E suggests that this isn't true - some pesticides banned in the US aren't banned in other countries, but SOME pesticides are banned in other countries that aren't banned in the US. Since there doesn't seem to be much rhyme or reason to which pesticides are banned and which aren't, maybe the fact that a pesticide is banned isn't the best indicator of its health risk, weakening the conclusion that these pesticides pose a health risk when they are consumed. Columnist assumes that banned pesticide -> health risk, so suggesting that this isn't true weakens argument. (I know I'm supposed to take the premises as facts, but I was under the impression that in 'weaken' questions, calling into question the validity of the premises is an effective way to undermine the argument.)

What have I missed in the stimulus that would've led me to C? Sorry I typed so much. I super appreciate that this is a thread.

Re: LSAT Q&A: Ask the Experts

Posted: Wed May 30, 2012 5:26 pm
by shifty_eyed
OMG how have I gotten so far in my LSAT prep without being able to tell the difference between sufficient and necessary assumption questions?

SMDH

Image
Jeffort, LSATblog, and bp shinners are the Richards sisters

I'm Taylor
Image

Re: LSAT Q&A: Ask the Experts

Posted: Wed May 30, 2012 8:58 pm
by deepblue
Would anyone be able to please explain clearly why the answer to PT08 RC (Section 3) Q#16 is A or B? The author doesn't seem skeptical to me. He seems to be opposed. Second, saying that A should be eliminated because of the broader term "literary analysis" is not a good reason. Look at lines 10 through 12.

Also, for question #18, do current tests test vocabulary like this one does? Had I known what "ostensible" means then perhaps I would have gotten this one. Context wasn't enough for me here. Just wondering if I have to focus on brushing up advanced vocabulary for the modern day test.

Also, for question #20, why B or D? I actually picked B and then crossed it out and wrote D. I guess the reason not to pick D is due to the word "illustrates." The passage is not using Blassingame's argument to simply illustrate a traditional approach but rather to attack it. So the word "mistaken" is the key here -- i.e. choice B.

Dunno why this passage messed me over.

Re: LSAT Q&A: Ask the Experts

Posted: Wed May 30, 2012 11:44 pm
by Manhattan LSAT Noah
shifty_eyed wrote:OMG how have I gotten so far in my LSAT prep without being able to tell the difference between sufficient and necessary assumption questions?
Not sure if that was a serious comment (I'm sort of distracted by that woman's face as she's sliding up and down the couch, so I can't read tone), but it's probably a good sign if you haven't delved into the difference and are getting assumption questions right. From my experience in training our teachers, most of them didn't think about the difference when they took the LSAT on their own and got a high score. It was only when they studied our curriculum that it started becoming something they considered. We teach the difference, but I do think it might get more hype than it deserves. Fun topic, though.

[edited in case a grammarian is watching]

Re: LSAT Q&A: Ask the Experts

Posted: Wed May 30, 2012 11:58 pm
by Manhattan LSAT Noah
cc.celina wrote:Thanks a bunch for answering my last question. Sadly I have another one: PT 47, Section 1, Q22 (pesticides).

Correct answer is C, but I picked E. How I went about it:

- It's a weaken question, so I noted the conclusion ("US exportation of banned pesticides greatly increases health risk to US consumers") and the assumption required to get to that conclusion: that some pesticides manufactured in the US are significantly harmful to the health US consumers when they subsequently eat products grown with those pesticides.

- Considered C, but I didn't see any reason to believe that, just because other countries manufacture these pesticides, means the ag. products on which they are used are exported to the US in particular. Even if they were, that doesn't render the US-manufactured pesticides any less harmful: the fact that a Smith & Wesson handgun (manufactured in the US) is dangerous to the safety of US citizens isn't changed by the fact that Glocks (manufactured in Austria) are also available. The combination of the two is more dangerous than the sole availability of either one, but that doesn't mean that either taken individually is harmless.
To me, C would only weaken the argument if the columnist had claimed, "Ceasing the practice of manufacturing banned pesticides in the US would completely eliminate the risk posed by banned pesticides." (Just as it would be unreasonable to conclude that, even if Smith & Wesson went out of business tomorrow and all their guns spontaneously self-destructed, nobody in the US would die from guns anymore - because Glocks are available as an alternative.) It didn't seem to have any bearing on the less forceful claim that dangerous US-manufactured pesticides are harmful when consumed.

- I didn't think E was a MUCH better answer, but I thought that it meant that the US wasn't really on top of its banning policy. The stimulus indicates that US banning policy is stricter than other countries, but E suggests that this isn't true - some pesticides banned in the US aren't banned in other countries, but SOME pesticides are banned in other countries that aren't banned in the US. Since there doesn't seem to be much rhyme or reason to which pesticides are banned and which aren't, maybe the fact that a pesticide is banned isn't the best indicator of its health risk, weakening the conclusion that these pesticides pose a health risk when they are consumed. Columnist assumes that banned pesticide -> health risk, so suggesting that this isn't true weakens argument. (I know I'm supposed to take the premises as facts, but I was under the impression that in 'weaken' questions, calling into question the validity of the premises is an effective way to undermine the argument.)

What have I missed in the stimulus that would've led me to C? Sorry I typed so much. I super appreciate that this is a thread.
Tough question!

First off, questioning the validity of a premise is not the LSAT's way of weakening an argument. You should read a few pages back in this thread and there's a discussion of this issue. It seems like that's happening in certain questions, but almost always (always?) it's really that the premise doesn't provide the information we think it does--often it's a study or something, and it's then treated as gospel.

In terms of this argument, the conclusion is that making and exporting banned pesticides greatly increases the risk to US consumers. Why? Because those pesticides are used on agricultural products that are sold to US.

You did identify a gap--is there a threat if you have these sprayed products imported? Couldn't it just be a threat to the folks around where the spraying occurs--but, as is often the case with tougher S/W questions, there's more than one gap. Here, there's also the assumption that the US not making and selling these pesticides would mean that they wouldn't be used on the products later shipped to the US. (C) plays on this. Sure, we're contributing, but is our practice greatly increasing the risk? Perhaps we're just the convenient provider of the risk, but others would step in. (C) doesn't take a lot of work to weaken.

(E), as I think you were seeing, takes a lot of gymnastics to become a weakener. There could be many reasons various pesticides are banned in some countries and not others. Off the top of my head, industry-lobbyists might get some competitor's pesticide banned. Or, religious reasons (we pray to DDT around here...)

So, I think you should takeaway from this question not to just go look for an answer that addresses the gap you saw, but instead get a sense of the argument and then work from wrong-to-right with a light touch, since you might not have seen another issue in the argument. (Gee, that got sort of poetic there.)

I hope that helps. There's a longer discussion of this question on our forums: http://www.manhattanlsat.com/forums/post2163.html

Re: LSAT Q&A: Ask the Experts

Posted: Thu May 31, 2012 10:56 am
by shifty_eyed
Manhattan LSAT Noah wrote:
shifty_eyed wrote:OMG how have I gotten so far in my LSAT prep without being able to tell the difference between sufficient and necessary assumption questions?
Not sure if that was a serious comment (I'm sort of distracted by that woman's face as she's sliding up and down the couch, so I can't read tone), but it's probably a good sign if you haven't delved into the difference and are getting assumption questions right. From my experience in training our teachers, most of them didn't think about the difference when they took the LSAT on their own and got a high score. It was only when they studied our curriculum that it started becoming something they considered. We teach the difference, but I do think it might get more hype than it deserves. Fun topic, though.

[edited in case a grammarian is watching]
SERIOUS!

Re: LSAT Q&A: Ask the Experts

Posted: Thu May 31, 2012 1:49 pm
by deepblue
Could someone clearly explain how I can differentiate between causal and conditional reasoning? Thanks!

Re: LSAT Q&A: Ask the Experts

Posted: Thu May 31, 2012 8:26 pm
by dkb17xzx
Thanks for doing this experts.

PT 61 Section 4, Q. 21 - It came down to C and D, and I picked D (incorrect). I notice that there is a shift from modern mammals to prehistoric marine reptiles. Is C a better answer because it bridges this and says that of both modern and prehistoric marine reptiles that were not deep divers, most had porous bones. So just b/c a prehistoric marine reptile has the same characteristic as a modern mammal, it doesn't mean it was able to do the same thing (deep dive) BECAUSE most this characteristic is common to the reptiles who are not deep divers?

I feel that D still weakens - but not as much as C.

Re: LSAT Q&A: Ask the Experts

Posted: Thu May 31, 2012 8:42 pm
by dkb17xzx
Also, PT 61, Section 4, # 24. Does "likely" affect the way a conditional reasoning is diagrammed?

My diagram:

Stranger one's approximate age --- > feel comfortable approaching stranger

~feel comfortable approaching stranger --> ~Stranger one's approximate age

= If one does not feel comfortable approaching a stranger, then that strangers is not of one's approximate age.

Is the argument assuming this:
~Stranger one's approximate age --> ~feel comfortable approaching stranger

This would be a mistaken reversal of the contrapositive.


And so E says, wait a minute - the argument has a mistaken reversal of the contrapositive of the original statement. We know that if you don;t feel comfortable, then you are not approaching a stranger your own age. But the argument never addresses whether someone feels comfortable approaching a stranger not of one's age?

Sorry if this is confusing

Re: LSAT Q&A: Ask the Experts

Posted: Thu May 31, 2012 9:25 pm
by cc.celina
Manhattan LSAT Noah wrote: I hope that helps. There's a longer discussion of this question on our forums: http://www.manhattanlsat.com/forums/post2163.html
Thanks a lot, your explanation and the discussion there really helped clear that up for me. For some reason i wasn't understanding that distinction between "greatly increased" and just plain "great."

Maybe this question is along similar lines: PT 32, Sec 1, Q 12 (polar bears).
I picked E, because I thought that if polar bears could smell familiar territory, that would go against one of the conditions necessary for it to qualify as navigation: it has to be "beyond the immediate range of the animal's senses." If the bear can smell the familiar territory, the territory isn't beyond the range of their senses.
I understand that B goes against the other, that it goes from "unfamiliar territory" to "points familiar to the animal."

I've been told that the right answer is 100% right and the other four are its logical opposite, and 100% wrong. Is E wrong because even though polar bears could smell familiar territory, it wasn't within the "immediate" range of their senses, just within plain old range?